$L= left( frac1rsintheta fracpartialpartial phi hatphi - frac1rfracpartial partial theta hattheta right) $

The name of the pictureThe name of the pictureThe name of the pictureClash Royale CLAN TAG#URR8PPP











up vote
1
down vote

favorite












Surely I'm making a trivial calculation error but I can not find it.



Let:
$$L(r,theta,phi)= frachbari r left( frac1rsintheta fracpartialpartial phi hatphi - frac1rfracpartial partial theta hattheta right)$$
$$L^2=Lcdot L= -hbar^2 r^2 left( frac1rsintheta fracpartialpartial phi hatphi - frac1rfracpartial partial theta hattheta right)^2=$$
$$= -hbar^2 r^2 left( frac1rsintheta fracpartialpartial phi hatphi - frac1rfracpartial partial theta hattheta right) cdot left( frac1rsintheta fracpartialpartial phi hatphi - frac1rfracpartial partial theta hattheta right)=$$
$$-hbar^2 left( frac1sin^2(theta) fracpartial^2partial phi^2 + fracpartial^2partial theta^2 right)$$



But I know that the correct result is:
$$L^2=-hbar^2 left( frac1sin^2(theta) fracpartial^2partial phi^2 + fracpartial^2partial theta^2 + cottheta fracpartialpartial theta right)$$







share|cite|improve this question


















  • 2




    The vectors $hatphi$ and $hattheta$ are also coordinate dependent.
    – HBR
    Aug 28 at 14:34






  • 1




    Also, you would be better off writing the diff'l operators 'last', e.g, $$ L= left( hatphifrac1rsintheta fracpartialpartial phi - hatthetafrac1rfracpartial partial theta right).$$ The way it was written originally, formally it looks like the operators are applied to the vectors. Reordering will make clearer that the product rule comes into play in the question.
    – peter a g
    Aug 28 at 14:57














up vote
1
down vote

favorite












Surely I'm making a trivial calculation error but I can not find it.



Let:
$$L(r,theta,phi)= frachbari r left( frac1rsintheta fracpartialpartial phi hatphi - frac1rfracpartial partial theta hattheta right)$$
$$L^2=Lcdot L= -hbar^2 r^2 left( frac1rsintheta fracpartialpartial phi hatphi - frac1rfracpartial partial theta hattheta right)^2=$$
$$= -hbar^2 r^2 left( frac1rsintheta fracpartialpartial phi hatphi - frac1rfracpartial partial theta hattheta right) cdot left( frac1rsintheta fracpartialpartial phi hatphi - frac1rfracpartial partial theta hattheta right)=$$
$$-hbar^2 left( frac1sin^2(theta) fracpartial^2partial phi^2 + fracpartial^2partial theta^2 right)$$



But I know that the correct result is:
$$L^2=-hbar^2 left( frac1sin^2(theta) fracpartial^2partial phi^2 + fracpartial^2partial theta^2 + cottheta fracpartialpartial theta right)$$







share|cite|improve this question


















  • 2




    The vectors $hatphi$ and $hattheta$ are also coordinate dependent.
    – HBR
    Aug 28 at 14:34






  • 1




    Also, you would be better off writing the diff'l operators 'last', e.g, $$ L= left( hatphifrac1rsintheta fracpartialpartial phi - hatthetafrac1rfracpartial partial theta right).$$ The way it was written originally, formally it looks like the operators are applied to the vectors. Reordering will make clearer that the product rule comes into play in the question.
    – peter a g
    Aug 28 at 14:57












up vote
1
down vote

favorite









up vote
1
down vote

favorite











Surely I'm making a trivial calculation error but I can not find it.



Let:
$$L(r,theta,phi)= frachbari r left( frac1rsintheta fracpartialpartial phi hatphi - frac1rfracpartial partial theta hattheta right)$$
$$L^2=Lcdot L= -hbar^2 r^2 left( frac1rsintheta fracpartialpartial phi hatphi - frac1rfracpartial partial theta hattheta right)^2=$$
$$= -hbar^2 r^2 left( frac1rsintheta fracpartialpartial phi hatphi - frac1rfracpartial partial theta hattheta right) cdot left( frac1rsintheta fracpartialpartial phi hatphi - frac1rfracpartial partial theta hattheta right)=$$
$$-hbar^2 left( frac1sin^2(theta) fracpartial^2partial phi^2 + fracpartial^2partial theta^2 right)$$



But I know that the correct result is:
$$L^2=-hbar^2 left( frac1sin^2(theta) fracpartial^2partial phi^2 + fracpartial^2partial theta^2 + cottheta fracpartialpartial theta right)$$







share|cite|improve this question














Surely I'm making a trivial calculation error but I can not find it.



Let:
$$L(r,theta,phi)= frachbari r left( frac1rsintheta fracpartialpartial phi hatphi - frac1rfracpartial partial theta hattheta right)$$
$$L^2=Lcdot L= -hbar^2 r^2 left( frac1rsintheta fracpartialpartial phi hatphi - frac1rfracpartial partial theta hattheta right)^2=$$
$$= -hbar^2 r^2 left( frac1rsintheta fracpartialpartial phi hatphi - frac1rfracpartial partial theta hattheta right) cdot left( frac1rsintheta fracpartialpartial phi hatphi - frac1rfracpartial partial theta hattheta right)=$$
$$-hbar^2 left( frac1sin^2(theta) fracpartial^2partial phi^2 + fracpartial^2partial theta^2 right)$$



But I know that the correct result is:
$$L^2=-hbar^2 left( frac1sin^2(theta) fracpartial^2partial phi^2 + fracpartial^2partial theta^2 + cottheta fracpartialpartial theta right)$$









share|cite|improve this question













share|cite|improve this question




share|cite|improve this question








edited Aug 28 at 15:40









Andrei

7,8802923




7,8802923










asked Aug 28 at 13:56









Stefano Barone

391110




391110







  • 2




    The vectors $hatphi$ and $hattheta$ are also coordinate dependent.
    – HBR
    Aug 28 at 14:34






  • 1




    Also, you would be better off writing the diff'l operators 'last', e.g, $$ L= left( hatphifrac1rsintheta fracpartialpartial phi - hatthetafrac1rfracpartial partial theta right).$$ The way it was written originally, formally it looks like the operators are applied to the vectors. Reordering will make clearer that the product rule comes into play in the question.
    – peter a g
    Aug 28 at 14:57












  • 2




    The vectors $hatphi$ and $hattheta$ are also coordinate dependent.
    – HBR
    Aug 28 at 14:34






  • 1




    Also, you would be better off writing the diff'l operators 'last', e.g, $$ L= left( hatphifrac1rsintheta fracpartialpartial phi - hatthetafrac1rfracpartial partial theta right).$$ The way it was written originally, formally it looks like the operators are applied to the vectors. Reordering will make clearer that the product rule comes into play in the question.
    – peter a g
    Aug 28 at 14:57







2




2




The vectors $hatphi$ and $hattheta$ are also coordinate dependent.
– HBR
Aug 28 at 14:34




The vectors $hatphi$ and $hattheta$ are also coordinate dependent.
– HBR
Aug 28 at 14:34




1




1




Also, you would be better off writing the diff'l operators 'last', e.g, $$ L= left( hatphifrac1rsintheta fracpartialpartial phi - hatthetafrac1rfracpartial partial theta right).$$ The way it was written originally, formally it looks like the operators are applied to the vectors. Reordering will make clearer that the product rule comes into play in the question.
– peter a g
Aug 28 at 14:57




Also, you would be better off writing the diff'l operators 'last', e.g, $$ L= left( hatphifrac1rsintheta fracpartialpartial phi - hatthetafrac1rfracpartial partial theta right).$$ The way it was written originally, formally it looks like the operators are applied to the vectors. Reordering will make clearer that the product rule comes into play in the question.
– peter a g
Aug 28 at 14:57










1 Answer
1






active

oldest

votes

















up vote
2
down vote



accepted










The problem is that if you change $theta$, the direction of $hatphi$ changes. You should be able to prove that $$fracpartial hatthetapartial phi=hatphicostheta$$
If you plug this into your equation, you will get the missing term.






share|cite|improve this answer




















  • Much easier to calculate $L_x$, $L_y$, and $L_z$ in terms of spherical coordinates
    – Andrei
    Aug 28 at 14:43










  • And also the coefficient of $hat phi$ depends on $theta$. The partial wrt $theta$ will act on that too (i.e., I mean it cannot be treated as a constant wrt $theta$).
    – peter a g
    Aug 28 at 14:45











  • Correct, but then you are left with $hatthetacdothatphi=0$ for that term
    – Andrei
    Aug 28 at 14:47










  • Yes - but the OP should notice that!
    – peter a g
    Aug 28 at 14:47










Your Answer




StackExchange.ifUsing("editor", function ()
return StackExchange.using("mathjaxEditing", function ()
StackExchange.MarkdownEditor.creationCallbacks.add(function (editor, postfix)
StackExchange.mathjaxEditing.prepareWmdForMathJax(editor, postfix, [["$", "$"], ["\\(","\\)"]]);
);
);
, "mathjax-editing");

StackExchange.ready(function()
var channelOptions =
tags: "".split(" "),
id: "69"
;
initTagRenderer("".split(" "), "".split(" "), channelOptions);

StackExchange.using("externalEditor", function()
// Have to fire editor after snippets, if snippets enabled
if (StackExchange.settings.snippets.snippetsEnabled)
StackExchange.using("snippets", function()
createEditor();
);

else
createEditor();

);

function createEditor()
StackExchange.prepareEditor(
heartbeatType: 'answer',
convertImagesToLinks: true,
noModals: false,
showLowRepImageUploadWarning: true,
reputationToPostImages: 10,
bindNavPrevention: true,
postfix: "",
noCode: true, onDemand: true,
discardSelector: ".discard-answer"
,immediatelyShowMarkdownHelp:true
);



);













 

draft saved


draft discarded


















StackExchange.ready(
function ()
StackExchange.openid.initPostLogin('.new-post-login', 'https%3a%2f%2fmath.stackexchange.com%2fquestions%2f2897291%2fl-left-frac1r-sin-theta-frac-partial-partial-phi-hat-phi-fr%23new-answer', 'question_page');

);

Post as a guest






























1 Answer
1






active

oldest

votes








1 Answer
1






active

oldest

votes









active

oldest

votes






active

oldest

votes








up vote
2
down vote



accepted










The problem is that if you change $theta$, the direction of $hatphi$ changes. You should be able to prove that $$fracpartial hatthetapartial phi=hatphicostheta$$
If you plug this into your equation, you will get the missing term.






share|cite|improve this answer




















  • Much easier to calculate $L_x$, $L_y$, and $L_z$ in terms of spherical coordinates
    – Andrei
    Aug 28 at 14:43










  • And also the coefficient of $hat phi$ depends on $theta$. The partial wrt $theta$ will act on that too (i.e., I mean it cannot be treated as a constant wrt $theta$).
    – peter a g
    Aug 28 at 14:45











  • Correct, but then you are left with $hatthetacdothatphi=0$ for that term
    – Andrei
    Aug 28 at 14:47










  • Yes - but the OP should notice that!
    – peter a g
    Aug 28 at 14:47














up vote
2
down vote



accepted










The problem is that if you change $theta$, the direction of $hatphi$ changes. You should be able to prove that $$fracpartial hatthetapartial phi=hatphicostheta$$
If you plug this into your equation, you will get the missing term.






share|cite|improve this answer




















  • Much easier to calculate $L_x$, $L_y$, and $L_z$ in terms of spherical coordinates
    – Andrei
    Aug 28 at 14:43










  • And also the coefficient of $hat phi$ depends on $theta$. The partial wrt $theta$ will act on that too (i.e., I mean it cannot be treated as a constant wrt $theta$).
    – peter a g
    Aug 28 at 14:45











  • Correct, but then you are left with $hatthetacdothatphi=0$ for that term
    – Andrei
    Aug 28 at 14:47










  • Yes - but the OP should notice that!
    – peter a g
    Aug 28 at 14:47












up vote
2
down vote



accepted







up vote
2
down vote



accepted






The problem is that if you change $theta$, the direction of $hatphi$ changes. You should be able to prove that $$fracpartial hatthetapartial phi=hatphicostheta$$
If you plug this into your equation, you will get the missing term.






share|cite|improve this answer












The problem is that if you change $theta$, the direction of $hatphi$ changes. You should be able to prove that $$fracpartial hatthetapartial phi=hatphicostheta$$
If you plug this into your equation, you will get the missing term.







share|cite|improve this answer












share|cite|improve this answer



share|cite|improve this answer










answered Aug 28 at 14:42









Andrei

7,8802923




7,8802923











  • Much easier to calculate $L_x$, $L_y$, and $L_z$ in terms of spherical coordinates
    – Andrei
    Aug 28 at 14:43










  • And also the coefficient of $hat phi$ depends on $theta$. The partial wrt $theta$ will act on that too (i.e., I mean it cannot be treated as a constant wrt $theta$).
    – peter a g
    Aug 28 at 14:45











  • Correct, but then you are left with $hatthetacdothatphi=0$ for that term
    – Andrei
    Aug 28 at 14:47










  • Yes - but the OP should notice that!
    – peter a g
    Aug 28 at 14:47
















  • Much easier to calculate $L_x$, $L_y$, and $L_z$ in terms of spherical coordinates
    – Andrei
    Aug 28 at 14:43










  • And also the coefficient of $hat phi$ depends on $theta$. The partial wrt $theta$ will act on that too (i.e., I mean it cannot be treated as a constant wrt $theta$).
    – peter a g
    Aug 28 at 14:45











  • Correct, but then you are left with $hatthetacdothatphi=0$ for that term
    – Andrei
    Aug 28 at 14:47










  • Yes - but the OP should notice that!
    – peter a g
    Aug 28 at 14:47















Much easier to calculate $L_x$, $L_y$, and $L_z$ in terms of spherical coordinates
– Andrei
Aug 28 at 14:43




Much easier to calculate $L_x$, $L_y$, and $L_z$ in terms of spherical coordinates
– Andrei
Aug 28 at 14:43












And also the coefficient of $hat phi$ depends on $theta$. The partial wrt $theta$ will act on that too (i.e., I mean it cannot be treated as a constant wrt $theta$).
– peter a g
Aug 28 at 14:45





And also the coefficient of $hat phi$ depends on $theta$. The partial wrt $theta$ will act on that too (i.e., I mean it cannot be treated as a constant wrt $theta$).
– peter a g
Aug 28 at 14:45













Correct, but then you are left with $hatthetacdothatphi=0$ for that term
– Andrei
Aug 28 at 14:47




Correct, but then you are left with $hatthetacdothatphi=0$ for that term
– Andrei
Aug 28 at 14:47












Yes - but the OP should notice that!
– peter a g
Aug 28 at 14:47




Yes - but the OP should notice that!
– peter a g
Aug 28 at 14:47

















 

draft saved


draft discarded















































 


draft saved


draft discarded














StackExchange.ready(
function ()
StackExchange.openid.initPostLogin('.new-post-login', 'https%3a%2f%2fmath.stackexchange.com%2fquestions%2f2897291%2fl-left-frac1r-sin-theta-frac-partial-partial-phi-hat-phi-fr%23new-answer', 'question_page');

);

Post as a guest













































































這個網誌中的熱門文章

How to combine Bézier curves to a surface?

Why am i infinitely getting the same tweet with the Twitter Search API?

Is there any way to eliminate the singular point to solve this integral by hand or by approximations?